Why is D wrong?
Hello. I narrowed it down to D and E but I chose E thinking that it was an alternative explanatio...
chelseaborg on December 22, 2019
  • September 2017 LSAT
  • SEC4
  • Q22
1
Reply
D vs E
I was torn between D and E and eventually went for E. Can someone please explain?
Minerva on December 22, 2019
  • September 2017 LSAT
  • SEC4
  • Q21
1
Reply
Why is C necessary?
I picked E. I realize E is wrong because vocabularly makes it not necessary. If I had picked up o...
Hunter on December 22, 2019
  • September 2017 LSAT
  • SEC4
  • Q18
1
Reply
Why is the answer C?
Why not E?
GLEE on December 22, 2019
  • September 2017 LSAT
  • SEC4
  • Q16
4
Replies
Explain
Hi can you explain why A is correct? Thank you!
Irene-Vera on December 22, 2019
  • September 2017 LSAT
  • SEC4
  • Q13
2
Replies
How can we Eliminate E
After reading the passage again, I get why the answer is D, BUT I still like E for an answer. How...
lerondagates on November 16, 2019
  • September 2017 LSAT
  • SEC4
  • Q21
2
Replies
Need help explaining the answer
Thanks.
cjahangiri on October 16, 2019
  • September 2017 LSAT
  • SEC4
  • Q18
1
Reply
Help
Can you please go through this question
odonnell on October 13, 2019
  • September 2017 LSAT
  • SEC4
  • Q12
1
Reply
Help
Is this a type of ad hominem reasoning?
@chris_va on October 1, 2019
  • September 2017 LSAT
  • SEC4
  • Q12
1
Reply
Why is E correct? Why is C incorrect?
Thanks
Ryan-Mahabir on September 28, 2019
  • September 2017 LSAT
  • SEC4
  • Q26
1
Reply
Why is D correct? Why is C incorrect?
Thanks
Ryan-Mahabir on September 24, 2019
  • September 2017 LSAT
  • SEC4
  • Q2
1
Reply
Why is D correct? Why is B incorrect?
Thanks
Ryan-Mahabir on September 24, 2019
  • September 2017 LSAT
  • SEC4
  • Q21
1
Reply
Why is C correct? Why is E incorrect?
Thanks
Ryan-Mahabir on September 13, 2019
  • September 2017 LSAT
  • SEC4
  • Q17
1
Reply
Why is A correct? Why is D incorrect?
Thanks
Ryan-Mahabir on September 13, 2019
  • September 2017 LSAT
  • SEC4
  • Q19
3
Replies
Why is E correct? Why is B incorrect?
Thanks
Ryan-Mahabir on September 13, 2019
  • September 2017 LSAT
  • SEC4
  • Q22
1
Reply
Option A
Can someone please explain why A is incorrect? Thanks!
Minerva on August 27, 2019
  • September 2017 LSAT
  • SEC4
  • Q9
1
Reply
Option A
Can someone please explain why A is correct? Thanks!
Minerva on August 26, 2019
  • September 2017 LSAT
  • SEC4
  • Q7
1
Reply
Question Stem Least Strengthens
The trouble I found came from the question stem as I was unsure what the criteria was for the cor...
Miketbrooks on August 24, 2019
  • September 2017 LSAT
  • SEC4
  • Q20
1
Reply
Correct but D
I got the answer correct however I was wondering why D is also incorrect. I have a feeling this a...
Miketbrooks on August 22, 2019
  • September 2017 LSAT
  • SEC4
  • Q3
1
Reply
Why is D incorrect?
Why is D incorrect?
Shiyi-Zhang on July 13, 2019
  • September 2017 LSAT
  • SEC4
  • Q13
4
Replies